0
$\begingroup$

Context: Solid state physics. Monoatomic linear chain.

Question: To prove that the total momentum of the chain is zero.

Attempted solution: I consider the sum:

\begin{align*} p = \sum_{n=1}^{N} m \dot{u_n} \end{align*}

where $u_n$ is the displacement from the equilibrium position of the $n$-th atom. The displacement is given by the formula:

\begin{align*} u_n = u_0 \exp\left[-i\left(\omega t \pm k n a\right)\right] \end{align*}

where $k$ is the wavevector, $n$ is the $n$-th atom and $a$ is the distance between atoms. If I substitute the above formula to the first sum, I get a result of the form:

\begin{align*} p \sim \sum_{n=1}^{N} \exp(i k n a) \end{align*}

I wonder, how could I prove that this is always zero ? If I treat it as the sum of a geometric series with $\alpha_1=\exp(ika)$ and $\lambda=\exp(ika)$, I still get a result that isn't necessarily zero.

\begin{align*} S_{1\to N} = \alpha_1 \frac{\lambda^N-1}{\lambda - 1} = e^{ika} \frac{e^{ikNa}-1}{e^{ika}-1} \end{align*}

If I further require that the first and last atoms are fixed, then $\exp(ikNa) = \exp(ika)$ and $S_{1\to N} = \exp(ika)$. Then

\begin{align*} p = -i\omega u_0 m \exp(-i\omega t) \exp(ika) = -i\omega m \underbrace{u_0 \exp\left[-i(\omega t - k a)\right]}_{u_1} = -i \omega m u_{1} = 0 \end{align*}

since we assumed that the 1st atom is fixed. Does this sound correct ?

$\endgroup$
1
  • $\begingroup$ 'If I treat it as the sum of the geometric series...' can you show this? $\endgroup$
    – Kvothe
    Apr 10, 2014 at 21:03

1 Answer 1

1
$\begingroup$

With $$u(n)=u_0e^{-i(\omega t+k n a)}$$ we have $$p=\sum_{n=1}^Nm\frac{d}{dt}u(n)=i\omega m u_0e^{-i\omega t}\left(\frac{1-e^{-iakN}}{1-e^{iak}}\right).$$ With cyclic boundary conditions we have $$u(N+1)=u(1)\Rightarrow k=\frac{2\pi j}{Na}\text{ for }j\in\mathbb{Z}$$ and inserting $k$ into $p$ gives $$p=i\omega m u_0e^{-i\omega t}\left(\frac{1-e^{-2 i \pi j}}{1-e^{\frac{2 i \pi j}{N}}}\right)=0.$$ Your method uses a slightly different boundary condition, but I think it's still valid.

$\endgroup$
0

Your Answer

By clicking “Post Your Answer”, you agree to our terms of service and acknowledge you have read our privacy policy.

Not the answer you're looking for? Browse other questions tagged or ask your own question.